Đến nội dung

Hình ảnh

Topic tổng hợp một số bất đẳng thức trong kì thi MO các nước

* * * * * 16 Bình chọn

  • Please log in to reply
Chủ đề này có 501 trả lời

#341
an1712

an1712

    Trung sĩ

  • Thành viên
  • 149 Bài viết

Bài 124:(IMO Shortlist) : Cho a,b,c là số đo 3 cạnh một tam giác. CMR:

 

$\frac{a^3}{b^2(c+a-b)}+\frac{b^3}{c^2(a+b-c)}+\frac{c^3}{a^2(b+c-a)}\geq 3$

áp dụng holder:

$(\sum \frac{a^3}{b^2(c+a-b)})(\sum bc+ab-b^2)(a+b+c)\geq (a+b+c)^3$

$bđt\Leftrightarrow \frac{(a+b+c)^2}{2(ab+bc+ac)-(a^2+b^2+c^2)}\geq 3$

$\sum a^2\geq \sum ab$ hiển nhiên


tiến tới thành công  :D


#342
dangkhuong

dangkhuong

    Sĩ quan

  • Thành viên
  • 312 Bài viết

Bài 126(IRAN 2009) : Cho $a,b,c >0$ thỏa mãn $a+b+c=3$ . CMR:

$\sum \frac{1}{a^2+b^2+2}\leq \frac{3}{4}$


Bài viết đã được chỉnh sửa nội dung bởi khanghaxuan: 20-06-2015 - 20:07

:ukliam2:  :ukliam2:  :ukliam2:


#343
dangkhuong

dangkhuong

    Sĩ quan

  • Thành viên
  • 312 Bài viết

Bài 126(IRAN 2009) : Cho a,b,c >0. CMR:nếu a+b+c=3

$\sum \frac{1}{a^2+b^2+2}\leq \frac{3}{4}$

Bài này thực sự là đặc chưng cho phương pháp tiếp tuyến, ta giải như sau:

1) Áp dụng bất đẳng thức AM-GM và giả thuyết thì chúng ta sẽ có BĐT đã cho tương đương BĐT sau:

$\sum \frac{1}{a^2-6a+13}\leq \frac{3}{8}$

Thật vậy lưu ý bất đẳng thức phụ sau:

$\frac{1}{a^2-6a+13}\leq \frac{a+1}{16}$

tương tự như thế suy ra$\sum \frac{1}{a^2-6a+13}\leq \frac{a+b+c+3}{16}=\frac{3}{8}$

từ đó ta có đpcm


Bài viết đã được chỉnh sửa nội dung bởi dangkhuong: 20-06-2015 - 15:56

:ukliam2:  :ukliam2:  :ukliam2:


#344
Nguyen Minh Hai

Nguyen Minh Hai

    Thiếu úy

  • Thành viên
  • 666 Bài viết

Bài 125:(Guzia MO) : Cho a,b,c >0: a+b+c=3. CMR:

 

$\sqrt[7]{\frac{a^3+2}{b(c+2a)}}+\sqrt[7]{\frac{b^3+2}{c(a+2b)}}+\sqrt[7]{\frac{c^3+2}{a(b+2c)}}\geq \frac{9abc}{a^2+b^2+c^2}$

Áp dụng $AM-GM$ cho vế trái ta được:

$VT \geq 3\sqrt[21]{\frac{(a^3+2)(b^3+2)(c^3+2)}{abc(a+2b)(b+2c)(c+2a)}}$                                    $(1)$

 

Theo BĐT $Holder$ ta lại có: $(a^3+2)(b^3+2)(c^3+2)\geq(a+b+c)^3=27$                                        $(2)$

 

Lại áp dụng BĐT $AM-GM$ ta được:

$abc(a+2b)(b+2c)(c+2a)\leq\left ( \frac{a+b+c}{3} \right )^3.\left ( \frac{3a+3b+3c}{3} \right )^3=27$  $(3)$

 

Từ $(1)$, $(2)$ và $(3)$ suy ra $VT \geq 3$

 

Mặt khác ta có: $a^2+b^2+c^2=\frac{1}{3}(a+b+c)(a^2+b^2+c^2)\geq 3abc$

$\Rightarrow VP=\frac{9abc}{a^2+b^2+c^2}\leq 3$

 

Do đó BĐT được chứng minh! 



#345
dangkhuong

dangkhuong

    Sĩ quan

  • Thành viên
  • 312 Bài viết

Bài 127(APMO Shortlist) : Cho a,b,c>0. CMR:

 

$\frac{a}{\sqrt{b+c-a}}+\frac{b}{\sqrt{c+a-b}}+\frac{c}{\sqrt{a+b-c}}\geq \sqrt{a}+\sqrt{b}+\sqrt{c}$


:ukliam2:  :ukliam2:  :ukliam2:


#346
dangkhuong

dangkhuong

    Sĩ quan

  • Thành viên
  • 312 Bài viết

Bài 128(Mở rộng khác của bài 118)(tự sáng tác): Cho a,b,c>0. CMR:

$\sqrt[4]{\frac{a^4+b^2+1}{21(b^2+c^2)+6ca}}+\sqrt[4]{\frac{b^4+c^2+1}{21(c^2+a^2)+6ca}}+\sqrt[4]{\frac{c^4+a^2+1}{21(a^2+b^2)+6ab}}\geq \frac{3}{2}$


Bài viết đã được chỉnh sửa nội dung bởi dangkhuong: 20-06-2015 - 16:13

:ukliam2:  :ukliam2:  :ukliam2:


#347
Nguyen Minh Hai

Nguyen Minh Hai

    Thiếu úy

  • Thành viên
  • 666 Bài viết

Bài 128(Mở rộng khác của bài 118)(tự sáng tác): Cho a,b,c>0. CMR:

$\sqrt[4]{\frac{a^4+b^2+1}{21(b^2+c^2)+6ca}}+\sqrt[4]{\frac{b^4+c^2+1}{21(c^2+a^2)+6ca}}+\sqrt[4]{\frac{c^4+a^2+1}{21(a^2+b^2)+6ab}}\geq \frac{3}{2}$

Hướng giải là tương tự bài 118

$AM-GM$ trực tiếp cho 3 số hạng vế trái và chú ý

$\prod(a^4+b^2+1)\geq(a^2+b^2+c^2)^3$

Còn cái dưới mẫu sau khi sử dụng $AM-GM$ ta tiếp tục đánh giá bằng $AM-GM$



#348
dangkhuong

dangkhuong

    Sĩ quan

  • Thành viên
  • 312 Bài viết

Bài 129(Putnam MO): Cho $a,b,c >0$. CMR nếu $abc=1$ thì:

$\frac{(2b+c)^3}{c^3(2b^3+a^3)^2}+\frac{(2c+a)^3}{a^3(2c^3+b^3)^2}+\frac{(2a+b)^3}{b^3(2a^3+c^3)^2}\leq 3(a^3+b^3+c^3)$


Bài viết đã được chỉnh sửa nội dung bởi khanghaxuan: 20-06-2015 - 20:08

:ukliam2:  :ukliam2:  :ukliam2:


#349
an1712

an1712

    Trung sĩ

  • Thành viên
  • 149 Bài viết

Bài 127(APMO Shortlist) : Cho a,b,c>0. CMR:

 

$\frac{a}{\sqrt{b+c-a}}+\frac{b}{\sqrt{c+a-b}}+\frac{c}{\sqrt{a+b-c}}\geq \sqrt{a}+\sqrt{b}+\sqrt{c}$

áp dụng holder:

$(\sum\frac{a}{\sqrt{b+c-a}})^2(\sum a(b+c-a)) \geq (a+b+c)^3$$bđt \Leftrightarrow \frac{(a+b+c)^3}{2(ab+bc+ac)-(a^2+b^2+c^2)}\geq \frac{3(a+b+c)^3}{(a+b+c)^2}=3(a+b+c)\geq (\sum \sqrt{a})^2$

=> đpcm


Bài viết đã được chỉnh sửa nội dung bởi an1712: 20-06-2015 - 16:28

tiến tới thành công  :D


#350
dangkhuong

dangkhuong

    Sĩ quan

  • Thành viên
  • 312 Bài viết

Bài 130(IMO 2001): Cho a,b,c>0. CMR:

$\sum \frac{a}{\sqrt{a^2+8bc}}\geq 1$


:ukliam2:  :ukliam2:  :ukliam2:


#351
dangkhuong

dangkhuong

    Sĩ quan

  • Thành viên
  • 312 Bài viết

áp dụng holder:

$(\sum\frac{a}{\sqrt{b+c-a}})^2(\sum a(b+c-a)) \geq (a+b+c)^3$$bđt \Leftrightarrow \frac{(a+b+c)^3}{2(ab+bc+ac)-(a^2+b^2+c^2)}\geq \frac{3(a+b+c)^3}{(a+b+c)^2}=3(a+b+c)\geq (\sum \sqrt{a})^2$

=> đpcm

Còn 1 cách khác cho bài toán này như sau:

Áp dụng bất đẳng thức Cauchy-Schawz ta đưa bài toán về chứng minh bất đẳng thức sau

$\sum \sqrt{a+b-c}\leq \sum \sqrt{a}$

Đặt $x= a+b-c; y=b+c-a; z=c+a-b$.

Khi đó bất đẳng thức trên tương đương bất đẳng thức sau đây:

$\sum \sqrt{\frac{x+y}{2}}\geq \sum \sqrt{x}$

Hiển nhiên đúng theo bất đẳng thức AM-GM. Vậy ta có đpcm.


Bài viết đã được chỉnh sửa nội dung bởi khanghaxuan: 20-06-2015 - 20:09

:ukliam2:  :ukliam2:  :ukliam2:


#352
dangkhuong

dangkhuong

    Sĩ quan

  • Thành viên
  • 312 Bài viết

Bài 131(Poland MO): Cho a,b,c>0. CMR

$\sqrt{\frac{a^5-a^2+3}{4b+5c}}+\sqrt{\frac{b^5-b^2+3}{4c+5a}}+\sqrt{\frac{c^5-c^2+3}{4a+5b}}\geq \sqrt{3}$


Bài viết đã được chỉnh sửa nội dung bởi dangkhuong: 20-06-2015 - 16:58

:ukliam2:  :ukliam2:  :ukliam2:


#353
dogsteven

dogsteven

    Đại úy

  • Thành viên
  • 1567 Bài viết

Bài 131(Poland MO): Cho a,b,c>0. CMR

$\sqrt{\frac{a^5-a^2+3}{4b+5c}}+\sqrt{\frac{b^5-b^2+3}{4c+5a}}+\sqrt{\frac{c^5-c^2+3}{4a+5b}}\geq 1$

$\prod (a^5-a^3+3)\geqslant \prod (a^2+2)\geqslant (a+b+c)^3$ còn $\prod (4b+5c)\leqslant 3^3(a+b+c)^3$

Do đó $VT\geqslant \sqrt{3}>1$


Quyết tâm off dài dài cày hình, số, tổ, rời rạc.


#354
dogsteven

dogsteven

    Đại úy

  • Thành viên
  • 1567 Bài viết

Bài 130(IMO 2001): Cho a,b,c>0. CMR:

$\sum \frac{a}{\sqrt{a^2+8bc}}\geq 1$

Có thể dùng Holder nhưng không thích, ta dùng dao mổ trâu :v

$x=\sqrt{\dfrac{a^2}{a^2+8bc}}, y=..., z=...$, ta cần chứng minh $x+y+z\geqslant 1$

Chú ý rằng $(1-x^2)(1-y^2)(1-z^2)=8^3x^2y^2z^2$, nếu $x+y+z<1$ thì:

$8^3x^2y^2z^2>(y+z)(z+x)(x+y)(y+z+2x)(z+x+2y)(x+y+2z)$

Bất đẳng thức hiển nhiên sai theo AM-GM


Bài viết đã được chỉnh sửa nội dung bởi dogsteven: 20-06-2015 - 17:00

Quyết tâm off dài dài cày hình, số, tổ, rời rạc.


#355
hoctrocuaHolmes

hoctrocuaHolmes

    Thượng úy

  • Thành viên
  • 1013 Bài viết

Bài 132:(IMO 1984) Cho $a,b,c>0$ thoả mãn $a+b+c=1$.Chứng minh rằng $0\leq ab+bc+ca-2abc\leq \frac{7}{27}$


Bài viết đã được chỉnh sửa nội dung bởi khanghaxuan: 20-06-2015 - 20:38


#356
Hoang Tung 126

Hoang Tung 126

    Thiếu tá

  • Thành viên
  • 2061 Bài viết

Bài 132:(IMO 1984) Cho $a,b,c>0$ thoả mãn $a+b+c=1$.Chứng minh rằng $0\leq ab+bc+ca-2abc\leq \frac{7}{27}$

   Ta có :$\frac{1}{a}+\frac{1}{b}+\frac{1}{c}\geq \frac{9}{a+b+c}=\frac{9}{1}=9$

$= > \frac{ab+bc+ac}{abc}\geq 9abc= > ab+bc+ac-2abc\geq 7abc> 0$

$= > ab+bc+ac-2abc> 0$  (1)

 

  Đồng nhất $a+b+c=1,(a+b+c)^3=1$

 

BDT $ab+bc+ac-2abc\leq \frac{7}{27}< = > (ab+bc+ac)(a+b+c)-2abc\leq \frac{7(a+b+c)^3}{27}$

$< = > 27ab(a+b)+27bc(b+c)+27ac(a+c)+27abc\leq 7(a+b+c)^3$

$< = > 27ab(a+b)+27bc(b+c)+27ac(a+c)+27abc\leq 7(a^3+b^3+c^3)+21ab(a+b)+21bc(b+c)+21ac(a+c)+42abc$

$< = > 7(a^3+b^3+c^3)+15abc\geq 6ab(a+b)+6bc(b+c)+6ac(a+c)$

 

 Theo BDT Schur bậc 3 ta có : $6(a^3+b^3+c^3+3abc)\geq 6(ab(a+b)+bc(b+c)+ac(a+c))$

 

 Theo Cosi ta có : $a^3+b^3+c^3\geq 3abc$

 

Cộng theo vế 2 BDT và rút gọn $= > 7(a^3+b^3+c^3)+15abc\geq 6ab(a+b)+6bc(b+c)+6ac(a+c)$ (2)

 

  Từ (1),(2) $= > 0< ab+bc+ac-2abc\leq \frac{7}{27}$ .Dấu = xảy ra khi $a=b=c=\frac{1}{3}$


Bài viết đã được chỉnh sửa nội dung bởi Hoang Tung 126: 20-06-2015 - 20:16


#357
Nguyenhuyen_AG

Nguyenhuyen_AG

    Trung úy

  • Thành viên nổi bật 2016
  • 945 Bài viết

Bài 133. Cho $a,\,b,\,c$ là ba số thực dương bất kỳ và đặt $k=abc.$ chứng minh rằng

\[\frac{1}{(a+1)^3}+\frac{1}{(b+1)^3}+\frac{1}{(c+1)^3} \geqslant \frac{-k^3+3k^2+k+1}{2(k+1)^3}.\]

Nếu $k=1$ thì ta được bất đẳng thức trong đề thi Việt Nam TST 2005.


Bài viết đã được chỉnh sửa nội dung bởi Nguyenhuyen_AG: 20-06-2015 - 21:07

Nguyen Van Huyen
Ho Chi Minh City University Of Transport

#358
Hoang Long Le

Hoang Long Le

    Thượng sĩ

  • Thành viên
  • 265 Bài viết

Bài 132:(IMO 1984) Cho $a,b,c>0$ thoả mãn $a+b+c=1$.Chứng minh rằng $0\leq ab+bc+ca-2abc\leq \frac{7}{27}$

 Cách khác cho í sau 

 Đặt $a=\frac{1}{3}+x~;b=\frac{1}{3}+y$ với $x,y\geq \frac{-1}{3}$

       $\Rightarrow c=\frac{1}{3}-x-y$ 

       $\Rightarrow ab+bc+ca-2abc=2xy(x+y)-\frac{x^2+y^2+xy}{3}+\frac{7}{27}$

        $=\frac{1}{12}.(6y-1)(2x+y)^2-\frac{1}{4}.y^2(2y+1)+\frac{7}{27}$

 Giả sử $x\geq y\Rightarrow \frac{1}{3}> x+y\geq 2y$

            $\Rightarrow y< \frac{1}{6}$

            $\Rightarrow \frac{1}{12}.(6y-1)(2x+y)^2-\frac{1}{4}.y^2(2y+1)\leq 0$

            $\Rightarrow ab+bc+ca-2abc\leq \frac{7}{27}$


IM LẶNG

#359
hoctrocuaHolmes

hoctrocuaHolmes

    Thượng úy

  • Thành viên
  • 1013 Bài viết

 Bài 90: (MOSP 2003) Cho 3 số thực không âm a,b,c thỏa mãn $a^2+b^2+c^2=1$ . Chứng minh rằng:

 

 $1\leq \sum \frac{a}{1+bc}\leq \frac{3\sqrt{3}}{4}$

Chứng minh vế thứ nhất: $1\leq \frac{a}{1+bc}+\frac{b}{1+ac}+\frac{c}{1+ab}$

Ta có $a+abc\leq a+\frac{a(b^{2}+c^{2})}{2}=a+\frac{a(1-a^{2})}{2}=\frac{-a^{3}+3a}{2}=1-\frac{a^{3}-3a+2}{2}=1-\frac{(a-1)^{2}(a+2)}{2}\leq 1\Leftrightarrow \frac{a^{2}}{a+abc}\geq a^{2}\Leftrightarrow \frac{a}{1+bc}\geq a^{2}$

Thiết lập các bất đẳng thức tương tự ta có $\frac{a}{1+bc}+\frac{b}{1+ac}+\frac{c}{1+ab}\geq a^{2}+b^{2}+c^{2}=1$  (theo gt)

Vậy bất đẳng thức đã được cm



#360
Hoang Long Le

Hoang Long Le

    Thượng sĩ

  • Thành viên
  • 265 Bài viết

Chứng minh vế thứ nhất: $1\leq \frac{a}{1+bc}+\frac{b}{1+ac}+\frac{c}{1+ab}$

Ta có $a+abc\leq a+\frac{a(b^{2}+c^{2})}{2}=a+\frac{a(1-a^{2})}{2}=\frac{-a^{3}+3a}{2}=1-\frac{a^{3}-3a+2}{2}=1-\frac{(a-1)^{2}(a+2)}{2}\leq 1\Leftrightarrow \frac{a^{2}}{a+abc}\geq a^{2}\Leftrightarrow \frac{a}{1+bc}\geq a^{2}$

Thiết lập các bất đẳng thức tương tự ta có $\frac{a}{1+bc}+\frac{b}{1+ac}+\frac{c}{1+ab}\geq a^{2}+b^{2}+c^{2}=1$  (theo gt)

Vậy bất đẳng thức đã được cm

 Bài này anh chứng minh ở đây rồi 

  - Trước tiên ta chứng minh $\sum \frac{a}{1+bc}\geq 1\Leftrightarrow \sum \frac{a^2}{a+abc}\geq a^2+b^2+c^2$

     Ta sẽ chứng minh $\frac{a^2}{a+abc}\geq a^2\Leftrightarrow a+abc\leq 1$

     Mà $a+abc\leq a+a.\frac{1-a^2}{2}\leq 1\Leftrightarrow (a+2)(a-1)^2\geq 0$ ( đúng )

           $\Rightarrow \sum \frac{a}{1+bc}\geq 1$

     Dấu "=" xảy ra khi $(a,b,c)=(1,0,0)$ và các hoán vị

   - Giờ ta chứng minh $\sum \frac{a}{1+bc}\leq \sqrt2$

     Ta sẽ chứng minh

         $\frac{a}{1+bc}\leq \frac{\sqrt2a}{a+b+c}$

         $\Leftrightarrow (a+b+c)^2\leq 2(1+bc)^2$

         $\Leftrightarrow \sum a^2+2\sum ab\leq 2+4bc+2b^2c^2$

         $\Leftrightarrow 1+2bc-2ac-2ab+2b^2c^2\geq 0$

         $\Leftrightarrow (b+c-a)^2+2b^2c^2\geq 0$ ( luôn đúng )

         $\Rightarrow \sum \frac{a}{1+bc}\leq \sqrt2$

       Dấu "=" xảy ra khi $(a,b,c)=(\frac{1}{\sqrt2},\frac{1}{\sqrt2},0)$ và các hoán vị.

 

Check


IM LẶNG




1 người đang xem chủ đề

0 thành viên, 1 khách, 0 thành viên ẩn danh